Sie sind auf Seite 1von 75

Solution to every third end-of-chapter problem

Basics of Engineering Economy, 2nd edition


Leland Blank and Anthony Tarquin

Chapter 1
Foundations of Engineering Economy
1.1

If the alternative that is actually the best one is not even recognized as an
alternative, it obviously will not be able to be selected by using any economic
analysis tools.

1.4

The analysis techniques that are used in engineering economic analysis are only
as good as the accuracy of the cash flow estimates.

1.7

In engineering economy, the evaluation criterion is financial units (dollars, pesos,


etc).

1.10

Time value of money means that there is a certain worth in having money and that
worth changes as a function of time.

1.13

The term that describes compensation for renting of money is time value of
money, which manifests itself as interest.

1.16

Minimum attractive rate of return is the lowest rate of return (interest rate) on a
project that companies or individuals consider to be high enough to induce them
to invest their money.

1.19

(a) Equivalent cost in 1 year = 38,000 + 38,000(0.10)


= $41,800
(b) Since $41,600 is less than $41,800, the firm should remodel 1ater (i.e. 1 year
from now).

1.22

Rate of return = (45/966)(100%)


= 4.7%

1.25

Rate of return = (2.3/6)(100%)


= 38.3%

1.28

Amount of earnings in year one = 400,000,000(0.25)


= $100,000,000

1.31

The engineer is wrong, unless the MARR is exactly equal to the cost of capital.
Usually, the inequality ROR MARR > cost of capital is used, and the MARR is
established higher than the cost of capital so that profit, risk and other factors are
considered.
1

1.34

F = P + Pni
100,000 = 1000 + 1000(n)(0.1)
99,000 = 1000(n)(0.10)
n = 990 years

1.37

P(1.20)(1.20) = 20,000
P = $13,888.89

1.40

F = P + P(n)(i)
3P = P + P(n)(0.20)
n = 10 years

1.43

All engineering economy problems will involve i and n

1.46

P = $50,000; F = ?; i = 15%; n = 3

1.49

F = $400,000; n = 2; i = 20% per year; P = ?

1.52

P = $16,000,000; A = $3,800,000; i = 18% per year; n = ?

1.55

The difference between cash inflows and cash outflows is known as net cash
flow.

1.58

Assuming down is negative: down arrow of $40,000 in year 5; up arrow in year 0


identified as P =?; i = 15% per year.

1.61

(a) FV is F (b) PMT is A

1.64

For built-in spreadsheet functions, a parameter that does not apply can be left
blank when it is not an interior one. For example, if no F is involved when using
the PMT function, it can be left blank because it is an end parameter. When the
parameter involved is an interior one (like P in the PMT function), a comma must
be put in its position.

1.67

(a) Assuming that Carols supervisor is a trustworthy and ethical person himself,
going to her supervisor and informing him of her suspicion is probably the
best of these options. This puts Carol on record (verbally) as questioning
something she heard at an informal gathering.
(b) Another good option is to go to Joe one-on-one and inform him of her
concern about what she heard him say at lunch. Joe may not be aware he is on
the bid evaluation team and the potential ethical consequences if he accepts
the free tickets from Dryer.

1.70

Answer is (a)

(c) NPER is n

(d) IRR is i

(e) PV is P

1.73

F = P(1+i)n
16,000 = 8000(1 + i)9
21/9 = 1 + i
1.08 = 1 + i
i = 0.08
(8%)
Answer is (b)

1.76

2P = P + P(n)(0.05)
1 = 0.05n
n = 20
Answer is (d)

Solution to every third end-of-chapter problem


Basics of Engineering Economy, 2nd edition
Leland Blank and Anthony Tarquin

Chapter 2
Factors: How Time and Interest Affect Money
2.1

(a) (F/P,10%,20) = 6.7275


(b) (A/F,4%,8) = 0.10853
(c) (P/A,8%,20) = 9.8181
(d) (A/P,20%,28) = 0.20122
(e) (F/A,30%,15) = 167.2863

2.4

(a) F = 885,000 + 100,000(F/P,10%,3)


= 885,000 + 100,000(1.3310)
= $1,018,000
(b) Spreadsheet function is = -FV(10%,3,,100000) + 885000
Display is $1,018,000

2.7

(a) F = 3000(F/P,10%,12) + 5000(F/P,10%,8)


= 3000(3.1384) + 5000(2.1436)
= $20,133.20
(b) Sum two calculator functions
FV(10,12,,-3000) + FV(10,8,-5000)
9,415.29 + 10,717.94 = $20,133.23
(c) If the spreadsheet function is = FV(10%,12,,3000) FV(10%,8,,5000), the
display is $20,133.23

2.10

A = 12,700,000(A/P,20%,8)
= 12,700,000(0.26061)
= $3,309,747

2.13

A = 20,000,000(A/P,10%,6)
= 20,000,000(0.22961)
= $4,592,200

2.16

(a) A = 3,000,000(10)(A/P,8%,10)
= 30,000,000(0.14903)
= $4,470,900
(b) If calculator function is PMT(8,10,-30000000,0), the answer is $4,470,884.66
(c) If the spreadsheet function is = -PMT(8%,10,30000000), display is
1

2.19

A = $4,470,884.66
(a) A = 225,000(A/P,15%,4)
= 225,000(0.35027)
= $78,811
(b) Recall amount = 78,811/0.10
= $788,110 per year

2.22

P = (280,000-90,000)(P/A,10%,5)
= 190,000(3.7908)
= $720,252

2.25

P = 2,100,000(P/F,10%,2)
= 2,100,000(0.8264)
= $1,735,440

2.28

P = 95,000,000(P/F,12%,3)
= 95,000,000(0.7118)
= $67,621,000

2.31

(a) P = 7000(P/F,10%,2) + 9000(P/F,10%,4) + 15,000(P/F,10%,5)


= 7000(0.8264) + 9000(0.6830) + 15,000(0.6209)
= $21,245.30
(b) Three calculator functions are added.
-PV(10,2,0,7000) PV(10,4,0,9000) PV(10,5,0,15000)
Total is 5785.12 + 6147.12 + 9313,82 = $21,246.06

2.34

A = 10,000,000(A/P,10%,10)
= 10,000,000(0.16275)
= $1,627,500

2.37

A = 3,250,000(A/P,15%,6)
= 3,250,000(0.26424)
= $858,780

2.40

A = 5000(7)(A/P,10%,10)
= 35,000(0.16275)
= $5696.25

2.43

(a) Let CF4 be the amount in year 4


100,000(F/P,9%,3) + 75,000(F/P,9%,2) + CF4(F/P,9%,1) = 290,000
100,000(1.2950) + 75,000(1.1881) + CF4(1.0900) = 290,000
(1.09)CF4 = 71.392.50
CF4 = $65,497.71
2

(b) F in year 5 for 2 known amounts


= -FV(9%,3,0,100000) - FV(9%,2,0,75000)
P in year 4 of $290,000 minus amount above (assume its in cell H9)
= -PV(9%,1,0,290000-H9)
Answer is $65,495.05
2.46

F = P(F/P,10%,n)
3P = P(F/P,10%,n)
(F/P,10%,n) = 3.000
From 10% interest tables, n is between 11 and 12 years. Therefore, n = 12 years

2.49

(a) P = 26,000(P/A,10%,5) + 2000(P/G,10%,5)


= 26,000(3.7908) + 2000(6.8618)
= $112,284
(b) Spreadsheet: enter each annual cost in adjacent cells and use the NPV
function to display P = $112,284
Calculators have no function for gradients; use the PV function on each cash
flow and add the five P values to get $112,284.55

2.52

A = 9000 560(A/G,10%,5)
= 9000 560(1.8101)
= $7986

2.55

A = 100,000 + 10,000(A/G,10%,5)
= 100,000 + 10,000(1.8101)
= $118,101
F = 118,101(F/A,10%,5)
= 118,101(6.1051)
= $721,018

2.58

475,000 = 25,000(P/A,10%,6) + G(P/G,10%,6)


475,000 = 25,000(4.3553) + G(9.6842)
9.6842G = 366,117.50
G = $37,805.65

2.61

A = 7,000,000 - 500,000(A/G,10%,5)
= 7,000,000 - 500,000(1.8101)
= $6,094,950

2.64

P = (23,000) 1 (1.02/1.10)5
(0.10 0.02)
= $90,405

2.67

First find P and then convert to A. (in million-people units)


P = 15,000(10)[1 (1.15/1.08)5]/(0.08 0.15)
= $790,491,225,000
A = 790,491,225,000(A/P,8%,5)
= 790,491,225,000(0.25046)
= $197.986 billion
(spreadsheet answer is $197,983,629,604)

2.70

Solve for P in geometric gradient equation and then convert to A


A1 = 5,000,000(0.01) = 50,000
P = 50,000[1 (1.10/1.08)5]/(0.08 0.10)
= $240,215
A = 240,215(A/P,8%,5)
= 240,215(0.25046)
= $60,164

2.73

Solve for A1 in geometric gradient equation and then find cost in year 3
400,000 = A1[1 (1.04/1.10)5]/(0.10 0.04)
4.0759 A1 = 400,000
A1 = $98,138
Cost in year 3 = 98,138(1.04)2
= $106,146

2.76

Since 4th deposit is known to be $1250, increase it by 5% each year to year one
A1 = 1250/(0.95)3
= $1457.94

2.79

F = 200,000(F/A,10%,6)
= 200,000(7.7156)
= $1,543,120

2.82

F in year 8 = 100(F/A,10%,3)(F/P,10%,6) + 200(F/A,10%,4)(F/P,10%,2)


= 100(3.3100)(1.7716) + 200(4.6410)(1.21)
= $1709.52

2.85

Find the future worth Fpaid of 3 payments in year 4


Fpaid = 2,000,000(F/A,8%,3)(F/P,8%,1)
= 2,000,000(3.2464)(1.08)
= $7,012,224
Find total amount owed Fowed after 4 years
Fowed = 10,000,000(F/P,8%,4)
= 10,000,000(1.3605)
= $13,606,000
Due in year 4 = 13,606,000 - 7,012,224
= $6,593,776

2.88

Find P in year 0 then convert to F. In $ million units,


P0 = 450 40(P/F,10%,1) + 200(P/A,10%,6)(P/F,10%,1)
= 450 40(0.9091) + 200(4.3553)(0.9091)
= $1205.52
F7 = 1205.52(F/P,10%,7)
= 1205.52(1.9487)
= $2349.20

2.91

Factors: (a) P = 31,000(P/A,8%,3) + 20,000(P/A,8%,5)(P/F,8%,3)


= 31,000(2.5771) + 20,000(3.9927)(0.7938)
= $143,278
(b) A = 143,278(A/P,8%,8)
= 143,278(0.17401)
= $24,932
Spreadsheet:

2.94

In $ billion units,
Gross revenue first 2 years = 5.8(0.701) = $4.0658
Gross revenue last 2 years = 6.2(0.701) = $4.3462
F = 4.0658(F/A,14%,2)(F/P,14%,2) + 4.3462(F/A,14%,2)
= 4.0658(2.1400)(1.2996) + 4.3462(2.1400)
= $20.6084 billion

2.97

First find F in year 8 and then solve for A


F8 = 15,000(F/A,8%,7) + 10,000(F/A,8%,4)
= 15,000(8.9228) + 10,000(4.5061)
= $178,903
A = 178,903(A/F,8%,8)
= 178,903(0.09401)
= $16,819

2.100 Find P in year -1for geometric gradient, than move to year 0 to find P
P-1 = (30,000) 1 (1.05/1.10)8
(0.10 0.05)
= $186,454
F = P0 = 186,454(F/P,10%,1)
= 186,454(1.10)
= $205,099
2.103 Find P in year 6 using arithmetic gradient factor and then find F today
P-6 = 10,000(P/A,12%,6) + 1000(P/G,12%,6)
= 10,000(4.1114) + 1000(8.9302)
= 41,114 + 8930.20
= $50,044.20
F = 50,044.20(F/P,12%,6)
= 122,439(1.9738)
= $98,777
2.106 (a) Add and subtract $2400 and $2600 in periods 3 and 4, respectively, to use
gradient
30,000 = 2000 + 200(A/G,10%,8) 2400(P/F,10%,3)(A/P,10%,8)
-2600(P/F,10%,4)(A/P,10%,8) + x(P/F,10%,3)(A/P,10%,8)
+ 2x(P/F,10%,4)(A/P,10%,8)

30,000 = 2000 + 200(3.0045) 2400(0.7513)( 0.18744)


-2600(0.6830)( 0.18744) + x(0.7513)(0.18744)
+ 2x(0.6830)( 0.18744)
30,000 = 2000 + 600.90 337.98 332.86 + 0.14082x + 0.25604x
0.39686x = 28,069.94
x = $70,730
(b) Spreadsheet uses Goal Seek to find x = $70,726

2.109 (a) Find P in year 4 for the geometric gradient,


then move all cash flows to future
P4 = 500,000[1 (1.15/1.12)16]/(0.12 0.15)
= $8,773,844
F = 500,000(F/A,12%,4)(F/P,12%,16) + P4(F/P,12%,16)
= 500,000(4.7793)(6.1304) + 8,773,844(6.1304)
= $68,436,684
2.112 Answer is (a)
2.115 A = 10,000,000((A/P,15%,7)
= $2,403,600
Answer is (a)
2.118 F = 50,000(F/P,18%,7)
= 50,000(3.1855)
= $159,275
Answer is (b)
2.121 10,000 = 2x(P/F,10%,2) + x(P/F,10%,4)
7

(b) Spreadsheet

10,000 = 2x(0.8264) + x(0.6830)


2.3358x = 10,000
x = $4281
Answer is (a)
2.124 1000(F/P,10%,20) + 1000(F/P,10%,n) = 8870
1000(6.7275) + 1000(F/P,10%,n) = 8870
1000(F/P,10%,n) = 2142.5
(F/P,10%,n) = 2.1425
n=8
Deposit year = 20 - 8 = 12
Answer is (d)

Solution to every third end-of-chapter problem


Basics of Engineering Economy, 2nd edition
Leland Blank and Anthony Tarquin

Chapter 3
Nominal and Effective Interest Rates
3.1

(a) Nom i/semi = 0.02*2 = 4% (b) Nom i/year = 0.02*4 = 8%


(c) Nom i/2 years = 0.02*8 = 16%

3.4

(a) year; (b) quarter; (c) day; (d) continuous (period length is zero); (e) hour

3.7

(a) r = 2% per quarter


(b) r = 2(2/3 quarter) = 1.33% per 2 months
(c) r = 2(8 quarters) = 16% per 2 years
(d) r = 2(4 quarters) = 8% per year
(e) r = 2(2 quarters) = 4% per semiannual period

3.10

(a) i/week = 6.8/26 = 0.262%; (b) effective

3.13

PP = day; CP = quarter

3.16

(a) Need effective i/quarter: i/quarter = 12%/4 = 3%


(b) Need effective i/semi: i/semi = (1 + 0.06/2)2 -1
= 0.0609 (6.09%)
(c) Need effective i/year: i/year = (1 + 0.12/4)4 -1
= 0.1255
(12.55%)

3.19

In $ million units,
F = (54 +14 + 10)(F/P,1.5%,12)
= 78(1.1956)
= $93.2568 ($93,256,800)

3.22

F = 242,000(F/P,1.5%,48)
= 242,000(2.0435)
= $494,527

3.25

P = 190,000(P/F,2%,8) + 120,000(P/F,2%,16)
= 190,000(0.8535) + 120,000(0.7284)
= $249,573

3.28

F = 18,000(F/P,8%,6) + 26,000(F/P,8%,5) + 42,000(F/P,8%,4)


= 18,000(1.5869) + 26,000(1.4693) + 42,000(1.3605)
= $123,907
1

3.31

P = 51(100,000)(0.25)(P/A,0.5%,60)
= (1,275,000)(51.7256)
= $65,950,140

3.34

First find P; then convert to A


P = 40,000 + 40,000(P/A,12%,2) + 50,000(P/A,12%,3)(P/F,12%,2)
= 40,000 + 40,000(1.6901) + 50,000(2.4018)(0.7972)
= $203,340
A = 203,340(A/P,12%,5)
= 203,340(0.27741)
= $56,409

3.37

i/week = 0.25%
P = 2.99(P/A,0.25%,40)
= 2.99(38.0199)
= $113.68

3.40

P = 90(P/A,3%,12) + 2.50(P/G,3%,12)
= 90(9.9540) + 2.50(51.2482)
= $1023.98

3.43

Find cost of treatments after one year, then monthly equivalent A over 5 years
Cost of treatment = 10,000(F/A,1%,12)
= 10,000(12.6825)
= $126,825
A = 126,825(A/P,1%,60)
= 126,825(0.02224)
= $2821 per month

3.46

i/quarter = (1 + 0.01)3 1 = 3.03%


P = [140,000 + 140,000(0.20)](P/A,3.03%,12)
= 168,000(9.9362)
= $1,669,282

3.49

F = 9000(F/A,1%,24)
= 9000(26.9735)
= $242,762

3.52

(a) A = 48,000 2000(P/G,1.5%,7)(P/F,1.5%,5)(A/P,1.5%,12)


= 48,000 2000(19.4018)(0.9283)(0.09168)
= $44,698
(b)

3.55

290,000(P/F,0.5%,48) = 4000(P/A,0.5%,48) + G(P/G,0.5%,48)


290,000(0.7871) = 4000(42.5803) + G(959.9188)
959.9188G = 57,938
G = $60.36

3.58

i = e0.0125 1 = 1.26% per month


F = 100,000(F/A,1.26%,24)
= 100,000{[1 + 0.0126)24 1]/0.0126}
= 100,000(27.8213)
= $2,782,130

3.61

A/3 months = 3(1500) = $4500


F = 4500(F/A,1.5%,20)
= 4500(23.1237)
= $104,057

3.64

Move chemical cost to end of interest period; find A


Chemical cost/month = 11(30) = $330
A = 1200(A/P,1%,48) + 330
= 1200(0.02633) + 330
= $361.60

3.67

Move cash flow to end of interest period (year); find P


Fuel savings = 800(0.50)(12) = $4800 per year
P = 4800(P/A,12%,3)
= 4800(2.4018)
= $11,529

3.70

Answer is (a)

3.73

P = 30(P/A,0.5%,60)
= $1552
Answer is (b)

3.76

PP > CP; must use i over PP of 1 year. Therefore, n = 7


Answer is (a)

Solution to every third end-of-chapter problem


Basics of Engineering Economy, 2nd edition
Leland Blank and Anthony Tarquin

Chapter 4
Present Worth Analysis
4.1

The do-nothing alternative is not an option (1) when it is absolutely required that
one of the defined alternatives be selected (such as for legal purposes), and (2)
when each alternative has only cost cash flow estimates.

4.4

(a) A, B and C are mutually exclusive; D and E are independent.


(b) X is in all bundles as the mutually exclusive selection. The two independent
projects have 22 = 4 bundles. The 4 viable options are:
X only

4.7

XD

XE

XDE

PW solar = -14,000 - 1500(P/A,10%,4) + 0.25(14,000)(P/F,10%,4)


= -14,000 - 1500(3.1699) + 3500(0.6830)
= $-16,364
PW line = -12,000 - 600(P/A,10%,4)
= -12,000 - 600(3.1699)
= $-13,902
Install power line; its cost is lower.

4.10

PWmanual = -425,000 [90,000(P/A,8%,5) + 7000(P/G,8%5)] + 80,000(P/F,8%,5)


= -425,000 [90,000(3.9927) + 7000(7.3724)] + 80,000(0.6806)
= $-781,502
PWrobotic = -850,000 [10,000(P/A,8%,5) + 1000(P/G,8%5)] + 300,000(P/F,8%,5)
= -850,000 [10,000(3.9927) + 1000(7.3724)] + 300,000(0.6806)
= $-693,119
Select the robotic system

4.13

Interest rate of 12% per year compounded monthly is 1% per month.


PW = -950 + 70(P/A,1%,36)
= -950 + 70(30.1075)
= $+1157.53
Since PW > 0, the service is financially justified

4.16

(a) PWA = -70,000 20,000(P/A,12%,3) + 15,000(P/F,12%,3)


= -70,000 20,000(2.4018) + 15,000(0.7118)
= $-107,359
1

PWB = -140,000 8,000(P/A,12%,3) + 40,000(P/F,12%,3)


= -140,000 8,000(2.4018) + 40,000(0.7118)
= $-130,742
Select method A
(b) A: Function is PV(12,3,-20000,15000) 70000
B: Function is PV(12,3,-8000,40000) 140000
4.19

PW = $-107,360
PW = $-130,743

(a) Monetary units are in $1000. Calculate PW values to select the pull system.
PWpull = -1500 700(P/A,10%,8) + 100(P/F,10%,8)
= -1500 700(5.3349) + 100(0.4665)
= $-5187.780 ($-5,187,780)
PWpush = -2250 600(P/A,10%,8) + 50(P/F,10%,8) 500(P/F,10%,3)
= -2250 600(5.3349) + 50(0.4665) 500(0.7513)
= $-5803.265 ($-5,803,265)
(b) By spreadsheet, enter the following into single cells to display the PW values.
PWpull : = - PV(10%,8,-700000,100000)-1500000
PWpush : = - PV(10%,8,-600000,50000)-2250000-PV(10%,3,,-500000)

4.22

(a) For Allison (A), use i = 1% per month and n = 60 months to calculate PW.
PWA = -40,000 - 5000(P/A,1%,60) + 10,000(P/F,1%,60)
= -40,000 - 5000(44.9550) + 10,000(0.5504)
= $-259,271
For Joshua (J), use effective semiannual i and n = 10 to calculate PW.
effective i = (1.01)6 -1 = 6.152%
PWJ = -60,000 13,000(P/A,6.152%,10) + 8,000(P/F,6.152%,10)
= -60,000 13,000(7.30737) + 8,000(0.55045)
= $-150,592
Select Joshuas plan

(b) A spreadsheet solution follows; select Joshuas plan

4.25

(a) Semiannual bond dividend is 1000(0.05)/2 = $25 per 6 months.


Semiannual interest rate is 5%/2 = 2.5%.
PW = -925 + 25(P/A,2.5%,16) + 800(P/F,2.5%,16)
= -925 + 25(13.0550) + 800(0.6736)
= $-59.74
No, the bond investment did not make the target rate since PW < 0.
(b) A spreadsheet solution follows to obtain PW = $-59.72

4.28

Semiannual bond dividend is 10,000(0.05)/2 = $250 per 6 months.


3

Semiannual interest rate expected is 6%/2 = 3%.


PW = -9000 + 250(P/A,3%,40) + 10,000(P/F,3%,40)
= -9000 + 250(23.1148) + 10,000(0.3066)
= $-155
No, the bond investment does not make the target rate since PW < 0.
A spreadsheet solution follows.

4.31

LCM is 48 months; repurchase P after 24 months


PWP = -13,650 - 200(P/A,1%,48) - 13,650(P/F,1%,24)
= -13,650 - 200(37.9740) - 13,650(0.7876)
= $-31,996
PWF = -22,900 - 50(P/A,1%,48) + 2000(P/F,1%,48)
= -22,900 - 50(37.9740) + 2000(0.6203)
= $-23,558
Select the fiber optic sensors

4.34

(a) LCM is 4 years; repurchased CFRP after 2 years


PWCFRP = -205,000 29,000(P/A,10%,4) 203,000(P/F,10%,2)
+ 2000(P/F,10%,4)
= -205,000 29,000(3.1699) 203,000(0.8264) + 2000(0.6830)
= $-463,320
PWFRC = -235,000 27,000(P/A,10%,4) + 20,000(P/F,10%,4)
= -235,000 27,000(3.1699) + 20,000(0.6830)
= $-306,927
Select material FRC

(b) PWCFRP function: = - 205000 - PV(10%,4,-29000,2000)+ PV(10%,2,,203000)


4

PWFRC function: = - 235000 - PV(10%,4,-27000,20000)


4.37

(a) PWC = -375,000 -200(P/A,8%,13)(P/F,8%,7)


= -375,000 200(7.9038)(0.5835)
= $-375,922
For asphalt, repave after 10 years and re-start maintenance charge in year 12.
PWA = -250,000[1+(P/F,8%,10)] 2500(P/A,8%,9)[(P/F,8%,1)+(P/F,8%,11)]
= -250,000[1.4632] 2500(6.2469)[0.9259 + 0.4289]
= $-386,958
Select the concrete option with a marginal advantage.
(b) Maintenance costs are incurred over 5 years only; there are none for concrete.
Now, select the asphalt option by a large margin.
PWC = $-375,000
PWA = -250,000 2500(P/A,8%,4)(P/F,8%,1)
= -250,000 2500(3.3121)(0.9259)
= $-257,667
A spreadsheet solution for parts (a) and (b) follows.

4.40

FWC = -80,000[(F/P,12%,15)+(F/P,12%,10)+(F/P,12%,5)+1]
= -80,000[5.4736 + 3.1058 + 1.7623 + 1]
= $-907,336
FWM = -200,000(F/P,12%,15) 500[(F/P,12%,12)+(F/P,12%,9)+(F/P,12%,6)
5

+(F/P,12%,3)+1] +25,000
= -200,000(5.4736) - 500[3.8969 + 2.7731 + 1.9738 + 1.4049 + 1] + 25,000
= $-1,075,244
Select the concrete exterior by a future worth amount of approximately $168,000.

4.43

LCCA = -750,000 (6000 + 2000)(P/A,0.5%,240) 150,000[(P/F,0.5%,60)


+ (P/F,0.5%,120) + (P/F,0.5%,180)]
= -750,000 (8000)(139.5808) 150,000[(0.7414) + (0.5496) + (0.4075)]
= $-2,121,421
LCCB = -1.1 (3000 + 1000)(P/A,0.5%,240)
= -1.1 (4000)(139.5808)
= $-1,658,323
Select proposal B.

4.46

CC = - [38(120,000) + 17(150,000)]/0.08
= $-88,875,000

4.49

CC = -1,700,000 350,000(A/F,6%,3)/0.06
= - 1,700,000 350,000(0.31411)/0.06
= $-3,532,308

4.52

CC = 100,000 + 100,000/0.08
= $1,350,000

4.55

Monetary terms are in $1000 units.


CC = -200 - 300(P/F,8%,4) - 50(A/F,8%,5)/0.08 - (8/0.08)(P/F,8%,14)
= -200 - 300(0.7350) - 50(0.17046)/0.08 - (8/0.08)(0.3405)
= $-561.088
($-561,088)

4.58

Quarterly interest rate is 12/4 = 3% with 4 quarters per year


6

CC = AW/i; select alternative E. Monetary values are in $1000 units.


CCE = [-2000(A/P,3%,16) + 300 + 50(A/F,3%,16)]/0.03
= [-2000(0.07961) + 300 + 50(0.04961)]/0.03
= $+4775.35
($+4,775,350)
CCF = [-3000(A/P,3%,32) + 100 + 70(A/F,3%,32)]/0.03
= [-3000(0.04905) + 100 + 70(0.01905)]/0.03
= $-1527.217
($-1,527,217)
CCG = -10,000 + 400/0.03
= $+3333.333
($+3,333,333)
By spreadsheet, enter the following into single cells to display the CC values
Answer: Select alternative E.
E: = (-PMT(3%,16,-2000000,50000)+300000)/0.03
F: = (-PMT(3%,32,-3000000,70000)+100000)/0.03
G: = -10000000+400000/0.03
4.61

Budget = $800,000

i = 10%

Display: $+4,775,295
Display: $-1,526,886
Display: $+3,333,333

6 viable bundles

Bundle

Projects

NCFj0

NCFjt

1
2
3
4
5
6

X
Y
Z
XY
XZ
Do nothing

$-250,000
-300,000
-550,000
-550,000
-800,000
0

$ 50,000
90,000
150,000
140,000
200,000
0

S
$ 45,000
-10,000
100,000
35,000
145,000
0

PW at 10%
$-60,770
-21,539
-6,215
-82,309
-66,985
0

PWj = NCFj(P/A,10%,4) + S(P/F,10%,4) - NCFj0


Since no bundle has PW > 0. Select Do nothing project.
4.64

P = -35,000 - 20,000(P/A,10%,4) - 25,000(P/F,10%,2) + 10,000(P/F,10%,4)


= -35,000 - 20,000(3.1699) - 25,000(0.8264) + 10,000(0.6830)
= $-112,228
Answer is (d)

4.67

CC = A/i
7

A = 10,000(A/F,10%,5)
= 10,000(0.16380)
= $-1638
CC = -1638/0.10
= $-16,380
Answer is (b)
4.70

Answer is (d)

Solution to every third end-of-chapter problem


Basics of Engineering Economy, 2nd edition
Leland Blank and Anthony Tarquin

Chapter 5
Annual Worth Analysis
5.1

AW = -7,000,000(A/P,15%,3) - 860,000
= -7,000,000(0.43798) - 860,000
= $-3,925,860
Therefore, required revenue is $3,925,860 per year

5.4

In $ millions,
AW = [-13 - 10(P/F,15%,1)](A/P,15%,10) 1.2
= [-13 - 10(0.8696)](0.19925) 1.2
= $-5.5229
Revenue required is $5,522,900 per year

5.7

First find PW, then annualize over three years


PW = -150,000[(400)(P/F,6%,1) + 300(P/F,6%,2) + 600(P/F,6%,3)]
= -60,000,000(0.9434) - 45,000,000(0.8900) - 90,000,000(0.8396)
= $-172,218,000
AW = -172,218,000(A/P,6%,3)
= -172,218,000(0.37411)
= $-64,428,476

5.10

(a) Use Equation [5.3] for CR per year.


CR = -3,800,000(A/P,12%,12) + 250,000(A/F,12%,12)
= -3,800,000(0.16144) + 250,000(0.04144)
= $-603,112
(b) AW = CR + A of AOC
= -603,112 350,000 25,000(A/G,12%,12)
= -603,112 350,000 - 25,000(4.1897)
= $-1,057,855
(c) One way to use a spreadsheet follows:
CR: single cell entry = -PMT(12%,12,-3800000,250000)
AW: Enter increasing gradient into B3:B14 and combine PMT functions
= -PMT(12%,12,-3800000,250000) - PMT(12%,12,NPV(12%,B3:B14))
1

5.13

(a) AWplastic = -0.90(110)(43,560)(A/P,8%,20)


= -4,312,440(0.10185)
= $-439,222
AWrubber = -2.20(110)(43,560)(A/P,8%,30)
= -10,541,520(0.08883)
= $-936,403
Select the plastic liner
(b)

Plastic: liner cost; 0.9(110)43560) = $4,312,440


Function: -PMT(8,20,0,-4312440) Display: AW = $-439,232
Rubber liner cost; 2.20(110)43560) = $10,541,520
Function: -PMT(8,30,0,-10541520) Display: AW = $-936,376
Select the plastic liner

5.16

Tabulated factor solution:


(a) Monetary terms in $ million. From AW, project clearly makes 10% per year.
AW = -170(A/P,10%,20) + 85(1-0.225)
= -170(0.11746) + 65.875
= $+45.90
($+45.9 million)
(b) AW goes negative between 30% and 40% per year.
@ 20%: AW = -170(A/P,20%,20) + 65.875 = $+30.96 million
@ 30%: AW = -170(A/P,30%,20) + 65.875 = $+14.60 million
@ 40%: AW = -170(A/P,40%,20) + 65.875 = $-2.21 million
Spreadsheet solution with x-y scatter chart:

5.19

(a) AW evaluation indicates chamber 490G to be more economic.


AWD103 = -400,000(A/P,10%,3) + 40,000(A/F,10%,3) 4000
= -400,000(0.40211) + 40,000(0.30211) 4000
= $-152,760
AW490G = -250,000(A/P,10%,2) + 25,000(A/F,10%,2) 3000
= -250,000(0.57619) + 25,000(0.47619) 3000
= $-135,143
(b) At P = $-300,000 and S = $30,000:
AWD103 = -300,000(A/P,10%,3) + 30,000(A/F,10%,3) 4000
= -300,000(0.40211) + 30,000(0.30211) 4000
= $-115,570
At P = $-500,000 and S = $50,000:
AWD103 = -500,000(A/P,10%,3) + 50,000(A/F,10%,3) 4000
= -500,000(0.40211) + 50,000(0.30211) 4000
= $-189,950
Cheaper model, P = $-300,000 will change the decision to D103.
(c)

By spreadsheet for part (b) use the PMT functions at different P values.
490G: = -PMT(10%,2,-250000,25000)-3000
Display: $-135,143
D103 @ P=-300,000: = -PMT(10%,3,-300000,30000)-4000
Display: $-115,571
D103 @ P=-500,000: = -PMT(10%,3,-500000,50000)-4000
Display: $-189,952

5.22

AW = PW(i)
AW = [5,000,000 + 2,000,000(P/F,10%,10) + 100,000/(0.10)(P/F,10%,10)](0.10)
= [5,000,000 + 2,000,000(0.3855) + 1,000,000(0.3855)](0.10)
= $615,650

5.25

(a) Determine amount needed at end of year 20, followed by A to accumulate


this future amount.
CC = P = A/i = 24,000/0.08 = $300,000
F = A(F/A,8%,20)
300,000 = A(45.7620)
A = $6556 per year
3

By spreadsheet, enter = -PMT(8%,20,,300000) to display $6556.


(b)

P = 24,000(P/A,8%,30)
= 24,000(11.2578)
= $270,187
270,187 = A(F/A,8%,20)
= A(45.7620)
A = $5904 per year
By spreadsheet, enter two functions.
P after 20 years: = -PV(8%,30,24000)
A over 20 years: = -PMT(8%,20,,270187)

Display: $270,187
Display: $5904

(c) For 30 year payout, difference = 6556 5904


= $652 per year less
5.28

Factors: Perpetual AW is equal to AW over one life cycle.


AW = {[-150,000(P/A,10%,4) - 25,000(P/G,10%,4)](P/F,10%,2)
- [225,000(P/A,10%,4)(P/F,10%,6)]}(A/P,10%,10)
= {[-150,000(3.1699) 25,000(4.3781)](0.8264)
-[225,000(3.1699)(0.5645)]}(0.16275)
= $-144,198
Spreadsheet

5.31

In $ million units. Effective annual i = (1.025)4 1 = 10.381%.


AWA = -10(A/P,10.381%,5) + 0.7(A/F,10.381%,5) 1.8
= -10(0.26637) + 0.7(0.16256) 1.8
= $-4.35
($-4.35 million)
AWB = -35(0.10381) 0.6
= $-4.23
($-4.233 million)
Select B

5.34

Answer is (b)

5.37

Answer is (d)

5.40

Perpetual AWA = -50,000(A/P,10%,3) 20,000 + 10,000(A/F,10%,3)


= -50,000(0.40211) 20,000 + 10,000(0.30211)
= $-37,084
AW is same for all years, including an infinite life.
Answer is (d)

Solution to every third end-of-chapter problem


Basics of Engineering Economy, 2nd edition
Leland Blank and Anthony Tarquin

Chapter 6
Rate of Return Analysis
6.1

(a) The highest possible is infinity


(b) The lowest possible is -100%

6.4

In $ million units
Annual payment = 6,000,000(A/P,10%,10)
= 6,000,000(0.16275)
= $976,500
Principal remaining after year 1 = 6,000,000(1.10) 976,500
= $5,623,500
Interest, year 2 = 5,623,500(0.10)
= $562,350

6.7

(a) Move all cash flows to year 1


0 = -80,000 + 9000(P/F,i*,1) + 70,000(P/F,i*,2) + 30,000(P/F,*i,3)
By trial and error, i* = 15.32%
(b) Enter cash flows for years 1 to 4 in cells B1:B4
Function = IRR(B1:B4) displays i* = 15.32%

6.10

(a) Tabulated factors


0 = -400,000(F/A,i*,10) + 270,000/i*
Try 5%: -400,000(12.5779) + 270,000/0.05 = $368,840
Try 6%: -400,000(13.1808) + 270,000/0.06 = $-772,320

too low
too high

Interpolation yields i* = 5.32%


(b) Solve for i* by spreadsheet using the FV function and GOAL SEEK. For
example, place a guess i* in cell B1and set up the single-cell function
= -FV($B$1,10,-400000) + 270000/$B$1
Display is i* = 5.29% per year
1

6.13

0 = -3,000,000 + 1,500,000(P/A,i*,3)
(P/A,i*,3) = 2.0000
Solve for i* = 23.38% per year

6.16

(spreadsheet)

0 = -65,220(P/A,i*,4) + (57,925 35,220)(P/A,i*,31)(P/F,i*,4)


= -65,220(P/A,i*,4) + (22,705)(P/A,i*,31)(P/F,i*,4)
Solve by trial and error:
Try 6%: 0 = -225,994 + 250,510 = $24,516 i too low
Try 7%: 0 = -220,913 + 217,071 = $-3842 i too high
i* = 6.86% per year

6.19

0 = 2,000,000 200,000(P/A,i*,2) 2,200,000(P/F,i*,3)


Solve for i* = 10% per year

6.22

(interpolation or spreadsheet)

(spreadsheet)

0 = -130,000 + (78,000 49,000)(P/A,i*,8) + 1000(P/G,i*,8) + 23,000(P/F,i*,8)


Solve for i* = 19.17 %

(spreadsheet)

6.25

(a) The rate of return on the increment has to be larger than 18%
(b) The rate of return on the increment has to be smaller than 10%
(c) Two samples follow with approximate ROR values of 10% for X
and 18% for Y

6.28

(a) Incremental CF, year 0: -25,000 (-15,000) = $-10,000


(b) Incremental CF, year 3: -400 - (-1600 15,000 + 3000) = $+13,200
(c) Incremental CF, year 6: (-400+6000) (-1600+3000) = $+4200

6.31

This is an incremental ROR analysis to find i*


0 = -250,000(F/P,i*,8) + 50,000(F/A,i*,8) + 30,000
Solve for i* = 13.14% < MARR = 15%

(spreadsheet)

Select P
6.34

Incremental CF amounts for (Y-X)


Incremental first cost = $-62,000
Incremental M&O = $3,000
Incremental revenue = $23,000
Incremental salvage = $7,000
0 = -62,000 + 3,000(P/A,i*,3) + 23,000(P/A i*,3) + 7000(P/F,i*,3)
Solve for i* by trial and error or spreadsheet
i* = 16.83% > MARR = 15%

(spreadsheet)

Select robot Y
6.37

Alternatives are independent; compare each against DN


Product 1:

0 = -340,000 + (180,000 70,000)(P/A,i*,5)


i* = 18.52% > MARR = 15%
Accept

Product 2:

0 = -500,000 + (190,000 64,000)(P/A,i*,5)


i* = 8.23% < MARR = 15%
Reject

Product 3:

0 = -570,000 + (220,000 48,000)(P/A,i*,5)


i* = 15.49% > MARR = 15%
Accept

0 = -620,000 + (205,000 40,000)(P/A,i*,5)


i* = 10.35% < MARR = 15%
Reject
Company should manufacture product lines 1 and 3
Product 4:

6.40

Rank revenue alternatives by increasing initial investment: DN, D, A, B, E, C


(a) MARR = 13.5%
DN to D: i* = 15% > MARR
A to D: i* = 12% < MARR
B to D: i* = 26% > MARR
E to B: i* = -16% < MARR
C to B: i* = 25% > MARR
Select C

eliminate DN
eliminate A
eliminate D
eliminate E
eliminate B

(b) MARR = 16%


D to DN: i* = 15% < MARR eliminate D
A to DN: i* = 13% < MARR eliminate A
B to DN: i* = 23% > MARR eliminate DN
E to B: i* = -16% < MARR eliminate E
C to B: i* = 25% > MARR eliminate B
Select C
6.43

(a) Select A and C with project ROR > MARR = 15.5%


(b) Rank proposals: DN, A, B, C, D; MARR = 10%
A to DN: i* = 29% < MARR eliminate DN
B to A: i* = 1% < MARR
eliminate B
C to A: i* = 7% < MARR
eliminate C
D to A: i* = 10% = MARR eliminate A

Select D

(c) Same as part (b) above, except last comparison against MARR = 14%
D to A: i* = 10% < MARR

eliminate D

Select A

6.46 (a) Two sign changes; maximum number of i* values is two


(b)
Year
0
1
2
3
4

Net Cash Flow, $


-40,000
32,000
18,000
-2000
-1000

Cumulative
Cash Flow, $
-40,000
-8000
+10,000
+8000
+7000

There is one sign change in the cumulative cash flow series; only one positive i*
value is indicated
6.49 Quarter Expenses
0
-20
1
-20
2
-10
3
-10
4
-10
5
-10
6
-15
7
-12
8
-15

Revenue
0
5
10
25
26
20
17
15
2

Net Cash Flow


-20
-15
0
15
16
10
2
3
-13

Cumulative
-20
-35
-35
-20
-4
+6
+8
+11
-2

(a) From net cash flows, there are two possible i* values
(b) From cumulative cash flow, sign starts negative and changes twice. Norstroms
criterion is not satisfied, there may be up to two i* values.
6.52

(a) 3 changes in sign of net cash flow; 3 possible i* values


(b)
Year
0
1
2
3
4
5

Net CF, $
-17,000
-20,000
4,000
-11,000
32,000
47,000

Cumulative
CF, $
-17,000
-37,000
-33,000
-44,000
-12,000
+35,000

Per Norstroms criterion, there is only one positive i* value


0 = -17 - 20(P/F,i*,1) + 4(P/F,i*,2) -11(P/F,i*,3) + 32(P/F,i*,4) + 47(P/F,i*,5)
i* = 17.39%
(spreadsheet)
6.55

Tabulate net cash flow and cumulative cash flow values

Year
0
1
2
3
4
5
6
7
8
9
10

Cash Flow, $
0
-5000
-5000
-5000
-5000
-5000
-5000
+9000
-5000
-5000
-5000 + 50,000

Cumulative
Cash Flow, $
0
-5,000
-10,000
-15,000
-20,000
-25,000
-30,000
-21,000
-26,000
-31,000
+14,000

(a) Cash flow rule of signs test (Descartes rule): three possible ROR values
Cumulative cash flow test (Norstroms criterion): one positive ROR value
(b) Move all cash flows to year 10 and solve for i*
0 = -5000(F/A,i*,10) + 14,000(F/P,i*,3) + 50,000
Solve for i* = 6.29%

(spreadsheet)

(c) Hand solution: MIRR with ir = 20% and ib = 8%


PW0 = -5000(P/A,ib,6) 5000(P/A,ib,3)(P/F,ib,7)
= -5000(P/A,8%,6) 5000(P/A,8%,3)(P/F,8%,7)
= -5000(4.6229) 5000(2.5771)(0.5835)
= $-30,631
FW10 = 9000(F/P,ir,3) + 50,000
= 9000(F/P,20%,3) + 50,000
= 9000(1.7280) + 50,000
= $65,552
Find ROR at which PW0 is equivalent to FW10
PW0(F/P,i,10) + FW10 = 0
-30,631(1 + i)10 + 65,552 = 0
(1 + i)10 = 2.1400
i = 7.91%
Spreadsheet solution: Enter cash flows for years 0 through 10 into cells
B2 through B12, The function = MIRR(B1:B11,8%,20%) displays
i = 7.90%
(d) Hand solution: In applying the ROIC procedure, all F values are negative
except the last one. Therefore, i is used in all equations. The ROIC EROR
(i) is the same as the internal ROR value (i*) of 6.29% per year.
Spreadsheet solution: Use Figure 6.7 as a model and apply GOAL SEEK to
obtain i = 6.29%

6.58 (a) Hand solution: ROIC procedure with ir = 14%


F0 = 5000
F1 = 5000(1.14) 2000 = 3700
F2 = 3700(1.14) 1500 = 2718
F3 = 2718(1.14) 7000 = -3901.48
F4 = -3901.48(1 + i) + 4000

F0 > 0; use ir
F1 > 0; use ir
F2 > 0; use ir
F3 < 0; use i

Set F4 = 0 and solve for i


i = 2.53%
Spreadsheet solution: ROIC with ir = 14% results in i = 2.53%
Before GOAL SEEK

After GOAL SEEK

(b) Hand solution: Apply MIRR procedure with ib = 7% and ir = 14%


PW0 = -2000(P/F,ib,1) 1500(P/F,ib,2) - 7000(P/F,ib,3)
= -2000(P/F,7%,1) 1500(P/F,7%,2) - 7000(P/F,7%,3)
= -2000(0.9346) 1500(0.8734) 7000(0.8163)
= $-8893
FW4 = 5000(F/P,ir,4) + 4000
= 5000(F/P,14%,4) + 4000
= 5000(1.6890) + 4000
= $12,445
Find EROR at which PW0 is equivalent to FW4
PW0(F/P,i,4) + FW4 = 0
-8893(1 + i)4 + 12,445 = 0
(1 + i)4 = 1.3994
i = 8.76%
Spreadsheet solution:
Enter cash flows in cells B2:B6 and apply function =MIRR(B2:B6,7%,14%) to
display i = 8.76%.
7

6.61

Answer is (b)

6.64

0 = -60,000 + 10,000(P/A,i*10)
(P/A,i*,10) = 6.0000
From tables, i* is between 10% and 11%
Answer is (a)

6.67

Answer is (c)

6.70

Answer is (c)

Solution to every third end-of-chapter problem


Basics of Engineering Economy, 2nd edition
Leland Blank and Anthony Tarquin

Chapter 7
Benefit/Cost Analysis and Public Sector Projects
7.1

It is best to take a limited viewpoint in determining benefits and disbenefits


Because, in the broadest sense, benefits and disbenefits will usually exactly offset each other.

7.4

Under a DBOMF contract, the contractor is responsible for managing the cash
flows for the project and the government unit remains responsible for obtaining
capital and operating funds for the project.

7.7

B/C = (2,740,000 380,000)/2,000,000


= 1.18

7.10

(a) AW of Costs = 13,000(A/P,10%,20) + 400


= 13,000(0.11746) + 400
= $1927
AW of B D = 3800 6750(A/F,10%,20)
= 3800 6750(0.01746)
= $3682
B/C = 3682/1927
= 1.91
(b) Let P = minimum first cost
AW of costs = P(A/P,10%,20) + 400 = 3682
P = (3682 400)/0.11746
= $27,941
The first cost must > $27,941,000 to force B/C < 1.0

7.13

AW of initial cost, C = 35,000,000(A/P,5%,30)


= 35,000,000(0.06505)
= $2,276,750
AW of B = AW of Benefits AW of disbenefits AW of M&O costs
= 6,500,000 - 1,700,000 - 900,000
= $3,900,000
Modified B/C = 3,900,000/2,276,750
= 1.71
1

7.16

Let P = initial cost


1.3 = 500,000/[P(A/P,7%,50) + 200,000]
1.3 = 500,000/[P(0.07246) + 200,000]
500,000 = 1.3[P(0.07246) + 200,000]
240,000 = (0.094198)P
P = $2,547,825

7.19

(a)

B = $600,000
D = $190,000
C = 650,000(A/P,6%,20) + 150,000
= 650,000(0.08718) + 150,000
= $206,667
B/C = (600,000 190,000)/206,667
= 1.98
Project is justified since B/C > 1.0

(b) Modified B/C = (600,000 190,000 150,000)/650,000(A/P.6%,20)


= 260,000/56,667
= 4.59
Project is justified since modified B/C > 1.0
7.22

B = 41,000(33 18) = $615,000


D = 1100(85) = $93,500
C = 750,000(A/P,0.5%,36)
= 750,000(0.03042)
= $22,815
B/C = (615,000 93,500)/22,815
= 22.86

7.25

Calculate AW of total cost and rank according to increasing cost.


AWpond = 58(A/P,6%,40) + 5.5
= 58(0.06646) + 5.5
= $9.35
AWexpand = 76(A/P,6%,40) + 5.3
= 76(0.06646) + 5.3
= $10.35
2

AWprimary = 2(A/P,6%,40) + 2.1


= 2(0.06646) + 2.1
= $2.23
AWpartial = 48(A/P,6%,40) + 4.4
= 48(0.06646) + 4.4
= $7.59
Benefits are directly estimated; DN is first alternative
Ranking is as follows: DN, Primary, Partial, Pond, Expand
Primary to DN:

B/C = 2.7/2.23
= 1.21
eliminate DN

Partial to Primary: B/C = (8.3-2.7)/(7.59 2.23)


= 1.04
eliminate Primary
Pond to Partial:

B/C = (11.1 - 8.3)/(9.35 - 7.59)


= 1.59
eliminate Partial

Expand to Pond:

B/C = (12.0 - 11.1)/(10.35 - 9.35)


= 0.90
eliminate Expand

Select the Pond System


7.28 Benefits are direct; determine AW of costs; order is DN, N, S
CN = 11,000,000(0.06) + 100,000 = $760,000 per year
CS = 27,000,000(0.06) + 90,000 = $1,710,000 per year
N to DN: B/C = (990,000-120,000)/760,000
= 1.14
eliminate DN
S to N: (B-D) = (2,100,000 300,000) (990,000 120,000) = $930,000
C = 1,710,000 760,000 = $950,000
B/C = 930,000/950,000 = 0.98
Select site N

eliminate S

7.31

Rank by increasing AW of total costs; order is: DN, EC, NS


AW of costs, EC = 38,000(A/P,7%,10) + 49,000
= 38,000(0.14238) + 49,000
= $54,410
AW of costs, NS = 87,000(A/P,7%,10) + 74,000
= 87,000(0.14238) + 74,000
= $86,387
EC to DN:

B/C = (B-D)/C = (110,000 26,000)/54,410


= 1.54
eliminate DN

NS to EC:
B = 130,000 110,000 = $20,000
D = 18,000 26,000 = $-8000
C = 86,387 - 54,410 = $31,977
B/C = [20,000 -(-8000)]/31,977
= 0.88

eliminate NS

Select method EC
7.34 Rank alternatives by increasing cost: DN, G, J, H, I, L, K
Eliminate H and K based on B/C to DN < 1.0
G to DN = 1.15
J to G = 1.07
I to J = 1.07
L to I = ?

eliminate DN
eliminate G
eliminate J

B/C for L-to- I comparison is not shown. Must compare L to I incrementally;


survivor is selected alternative.
7.37

Answer is (c)

7.40

Can use either PW, AW, or FW values; For PW,


B/C = (245,784 30,723)/(100,000 + 68,798) = 1.27
Answer is (a)

7.43

B/CX = (110,000 20,000)/(60,000 + 45,000)


= 0.86 reject X
B/CY = (150,000 45,000)/(90,000 + 35,000)
= 0.84 reject Y
Answer is (a)

7.46

In $1000 units,
B/C = [(220 140) (30 10)]/(450 - 300) = 0.40
Answer is (b)

7.49

B/C = (360,000 42,000)/[2,000,000(0.06)] = 2.65


Answer is (d)

Solution to every third end-of-chapter problem


Basics of Engineering Economy, 2nd edition
Leland Blank and Anthony Tarquin

Chapter 8
Breakeven, Sensitivity, and Payback Analysis
8.1

(a) QBE = 1,000,000/(9.90 4.50)


= 185,185 units
(b) Profit = R TC
= 9.90(150,000) 1,000,000 4.50(150,000)
= $-190,000 (loss)
(c) Profit = R TC
= 9.90(480,000) 1,000,000 4.50(480,000)
= $1,592,000

8.4

Let x = hours per month billed to realize a profit of $15,000


15,000 = -900,000(A/P,1%,120) 1,100,000 + 1,500,000(A/F,1%,120) + 10(90)x
15,000 = -900,000(0.01435) 1,100,000 + 1,500,000(0.00435) + 10(90)x
900x = 1,121,390
x = 1246 hours/month

8.7 (a) Plot shows maximum quantity at about 1350 units. Profit is about $20,000.

(b) Profit = R TC = (-0.007 - 0.004) Q2 + (32-2.2)Q - 8


= -0.011Q2 + 29.8Q - 8
Qp = -b/2a = -29.8/2(-0.011)
= 1355 units
1

Max profit = -b2/4a + c = -29.82 / 4(-.011) - 8


= $20,175 per month
8.10

(a) Using the relation PW = 0, select different i values and solve for n. Details for
i = 8% and 15% are shown.
0 = -3,150,000 + 500,000(P/A,i%,n) + 400,000(P/F,i%,n)
i = 8%, n = 8: PW = -3,150,000 + 500,000(5.7466) + 400,000(0.5403)
= $-60,580
i = 8%, n = 9: PW = -3,150,000 + 500,000(6.2469) + 400,000(0.5002)
= $173,530
n = 8.2 (interpolation)
i = 15%, n = 19: PW = -3,150,000 + 500,000(6.1982) + 400,000(0.0703)
= $-22,780
i = 15%, n = 20: PW = -3,150,000 + 500,000(6.2593) + 400,000(0.0611)
= $4090
n = 19.8 years
(interpolation)
(b) Retention ranges from 8 to 20 years for varying i values. This is a perfect
example where the spreadsheet is easier. Use the NPER function.

8.13

Let Pasp = maximum $/mile for asphalt


-2,300,000(A/P,8%,20) - 483 = -Pasp(A/P,8%,10) 774
-2,300,000(0.10185) - 483 = -Pasp(0.14903) 774
Pasp = $1,569,912

8.16

Let x = square yards per year to break even


-109,000 2.75x = -225,000(A/P,8%,15) 13x
-109,000 2.75x = -225,000(0.11683) 13x
10.25x = 82,713
x = 8070 square yards/year

8.19

Let x = ads per year


-21 x = -12,000(A/P,10%,3) -55,000 + 2000(A/F,10%,3) 5x
-21x = -12,000(0.40211) -55,000 + 2000(0.30211) 5x
16x = 59,221
x = 3701 ads

8.22

Let x = production in year 3


-40,000 50x = -70,000 12x
38x = 30,000
x = 789.5 or 790 units

8.25

(a) Let T = number of tons. Solve relation AW1 = AW2 for T.


Variable costs (VC) for each machine
VC1: 24T/10 = 2.4T
VC2: 2(24)T/6 = 8T
-123,000(A/P,7%,10) 5000 -2.4T = -70,000(A/P,7%.6) 2500 8T
(8-2.4)T = -70,000(0.20980) 2500 + 123,000(0.14238) + 5000
5.6T = 5327
T = 951.2 tons
If tonnage is less than breakeven, select machine 2 since the slope is steeper.
At 1000 tons, select machine 1.
(b) Set up the VC relation for each machine and solve for T in AW1 = AW2 .
Cost,
% change $/hour
-15%
20.4
-5%
22.8
0%
24.0
+5%
25.2
+15%
27.6

VC1
2.04T
2.28T
2.40T
2.52T
2.76T

VC2
6.8T
7.6T
8.0T
8.4T
9.2T

AW1 = AW2
Breakeven
relation
T value, tons
4.76T = 5327
1119
5.32T = 5327
1001
5.60T = 5327
951
5.88T = 5327
906
6.44T = 5327
827

SOLVER can be used to find breakeven with the constraint AW1 = AW2.

8.28

(a) Solve relation Revenue - Cost = 0 for Q = number of filters per year
50Q - [200,000(A/P,6%,5) + 25,000 + 20Q] = 0
30Q = 200,000(0.23740) +25,000
Q = 72,480/30
= 2416 filters per year
At 5000 units, make the filters inhouse
(b) Solve the relation AWbuy = AWmake
(50-30)Q = -72,480 + (50-20)Q
Q = 7248 filters per year
Since 5000 < 7248, outsourcing is the correct choice; slope of 30Q is larger
(c) Inhouse: make 5000 at $20 each
Profit = 5000(50-20) - 72,480 = $77,520
Outsource: buy 5000 at $30 each
Profit = 5000(50-30) = $100,000
A spreadsheet can be used to answer all three questions.

8.31

AWcont = -130,000(A/P,15%,5) -30,000 + 40,000(A/F,15%,5)


= -130,000(0.29832) -30,000 + 40,000(0.14832)
= $-62,849
Lowest cost for batch will occur when the interest rate is the lowest (5%)
and life is longest (10 years)
AWbatch = -80,000(A/P,5%,10) 55,000 + 10,000(A/F,5%,10)
= -80,000(0.12950) 55,000 + 10,000(0.07950)
= $-64,565
Since batch AW is more costly, the batch system will never be less expensive
than the continuous option.

8.34

AWcurrent = $-62,000
AW10,000 = - 64,000(A/P,15%,3) 38,000 + 10,000(A/F,15%,3)
= - 64,000(0.43798) 38,000 + 10,000(0.28798)
= $-63,151
AW13,000 = - 64,000(A/P,15%,3) 38,000 + 13,000(A/F,15%,3)
= - 64,000(0.43798) 38,000 + 13,000(0.28798)
= $-62,287
AW18,000 = - 64,000(A/P,15%,3) 38,000 + 18,000(A/F,15%,3)
= - 64,000(0.43798) 38,000 + 18,000(0.28798)
= $-60,847
The decision is sensitive; replace the existing process only if the estimate of
$18,000 is reliable.

8.37

(a) AW1 = -10,000(A/P,i%,8) - 600 - 100(A/F,i%,8) - 1750(P/F,i%,4)(A/P,i%,8)


AW2 = -17,000(A/P,i%,12) - 150 - 300(A/F,i%,12)
- 3000(P/F,i%,6)(A/P,i%,12)
Calculate AW at each MARR value. The decision is sensitive to MARR,
changing at i = 6%.
MARR
4%
6%
8%

AW1
$-2318
$-2444
$-2573

AW2
$-2234
$-2448
$-2673

Selection
2
1
1

(b) Spreadsheet solution requires that the PW value is first determined using the
NPV function over the LCM of 24 years and then converting it to an AW value
using the PMT function. Spreadsheet follows.

8.40

Hand solution: Determine AW values at different savings, s.


AWA = -50,000(A/P,10%,5) 7500 + 5,000(A/F,10%,5) + s
= -50,000(0.2638) 7500 + 5000(0.1638) + s
= -19,871 + s
AWB = -37,500(A/P,10%,5) 8000 + 3700(A/F,10%,5) + s
= -37,500(0.2638) 8000 + 3700(0.1638) + s
= -17,286 + s
Selection changes when s is +40% of best estimate. Table follows:

Percent Savings for A,


Savings for B,
variation
$ per year
AWA
$ per year
AWB Selection
-40%
9,000
$-10,871
7,800
$-9,486
B
-20
12,000
-7,871
10,400
-6,886
B
0
15,000
-4,871
13,000
-4,286
B
20
18,000
-1,871
15,600
-1,686
B
40
21,000
1,129
18,200
914
A
Spreadsheet solution: PMT functions display AW values with savings variation
added to end of function.

8.43

(a) Set up the general AW relation for D103 and determine AW for the three
scenarios.
AWD103 = -P(A/P,10%,n) + (0.1P)(A/F,10%,n) 4000
Strategy
P
S
n
AW
Pess
-500,000 50,000 1 $-504,000
ML
-400,000 40,000 3 -152,761
Opt
-300,000 30,000 5
-78,226
The AW490G = $-135,143 is known. D103 is selected only if the optimistic
scenario is correct, i.e., P = $-300,000 and n = 5 years.
(b) Spreadsheet solution.

8.46

(a)

Machine 1:
7

np = 40,000/10,000
= 4 years
Machine 2:
np = 90,000/15,000
= 6 years
Select only machine 1.
(b) Machine 1:
-40,000 + 10,000(P/A,10%,np) = 0
(P/A,10%,np) = 4.0000
From 10% interest tables, np is between 5 and 6 years; np > 5 years.
Machine 2:
-90,000 + 15,000(P/A,10%,np) = 0
(P/A,10%,np) = 6.0000
From 10% interest tables, np is between 9 and 10 years np > 9 years
Select neither alternative for further analysis
8.49

(a) Let np = number of months at 0.5% per month


0 = -90,000(A/P,0.5%,np) 20,000 + 22,000
(A/P,0.5%,np) = 2000/90,000
= 0.02222
From 0.5% interest factor table, np is approximately 51 months
(b) Calculator function n(0.5,2000,-90000,0) displays np = 51.1 months
(c) Spreadsheet function =NPER(0.5%,2000,-90000) displays np = 51.1 months

8.52 Set up the PW relation and use trial and error or spreadsheet for np.
0 = -10,000 +1700(P/A,8%,np) + 900(P/F,8%,np)
For np = 7:
For np = 8:

0 = -10,000 +1700(5.2064) + 900(0.5835) = $-624


0 = -10,000 +1700(5.7466) + 900(0.5403) = $+255
np = 7.7 years
(interpolation)

Lease; dont purchase


Spreadsheet: NPER(8%,1700,-10000,900) displays 7.7 years

8.55

(a) Determine i* from AW relations


AW1 = -50,000(A/P,i*,5) + 24,000
(A/P,i*,5) = 0.4800
i1* = 38.6%
(interpolated)
AW2 = -120,000(A/P,i*,10) + 42,000 2500(A/G,i*,10)
i2* = 26.55%
(trial and error or spreadsheet)
Now, select process 1
(b) First of all, an ROR analysis always requires an incremental analysis over the
LCM of 10 years. The i2* = 26.55% is not correctly used in the fashion
shown here. Furthermore, this analysis assumes a return of 26.55% is made
from years 6 through 10, which may not be correct for these funds.

8.58

Both the fixed cost and variable cost of alternative X are higher than those of
alternative Y. Therefore, it can never be favored.
Answer is (a)

8.61

Answer is (a)

8.64

Answer is (c)

8.67

-100Q = -250,000(A/P,15%,4) 80,000 40Q


60Q = 250,000(0.35027) + 80,000
Q = 2793
Larger than 2793 since slope of make item inhouse is lower
Answer is (b)

8.70

Select largest PW.


Answer is (d)

Solution to every third end-of-chapter problem


Basics of Engineering Economy, 2nd edition
Leland Blank and Anthony Tarquin

Chapter 9
Replacement and Retention Decisions
9.1

The defender refers to the currently-owned, in-place asset while the challenger
refers to the equipment/process that is under consideration as its replacement.

9.4

P = market value = $39,000


AOC = $17,000 per year
n = 4 years
S = $25,000
(Note: P and AOC will carry signs in an evaluation)

9.7

The ESL of the defender is 3 years with the lowest AW of $-85,000.

9.10

Add AW values for first cost, operating cost, and salvage value; select lowest AW.
Years Retained
1
2
3
4
5
Economic service life is n = 4 years.

9.13

Total AW
$-102,000
-84,334
-84,190
-83,857
-84,294

(a) Tabulated factors


AW1 = -65,000(A/P,10%,1) 50,000 + 30,000(A/F,10%,1) = $-91,500
AW2 = -65,000(A/P,10%,2) [50,000 + 10,000(A/G,10%,2)]
+ 30,000(A/F,10%,2)
= $-77,929
AW3 = -65,000(A/P,10%,3) [50,000 + 10,000(A/G,10%,3)]
+ 20,000(A/F,10%,3)
= $-79,461
AW4 = -65,000(A/P,10%,4) [50,000 + 10,000(A/G,10%,4)]
+ 20,000(A/F,10%,4)
= $-80,008
AW5 = -65,000(A/P,10%,5) [50,000 + 10,000(A/G,10%,5)]
+ 20,000(A/F,10%,5)
= $-81,972
1

AW6 = -65,000(A/P,10%,6) [50,000 + 10,000(A/G,10%,6)]


+ 20,000(A/F,10%,6)
= $-84,568
AW7 = -65,000(A/P,10%,7) [50,000 + 10,000(A/G,10%,7)]
+ 20,000(A/F,10%,7)
= $-87,459
ESL is n = 2 years with AW = $-77,929
(b) Spreadsheet solution shows ESL is n = 2 years with AW = $77,929

9.16

AWC = -80,000(A/P,12%,3) - 19,000 + 10,000(A/F,12%,3)


= -80,000(0.41635) - 19,000 + 10,000(0.29635)
= $-49,345
AWD = -10,000(A/P,12%,3) -15,000 31,000 + 9000(A/F,12%,3)
= -10,000(0.41635) -15,000 31,000 + 9000(0.29635)
= $-47,496
AWD = -20,000(A/P,12%,3) -15,000 31,000 + 9000(A/F,12%,3)
= -20,000(0.41635) -15,000 31,000 + 9000(0.29635)
= $-51,660
Trade-in of $10,000: Select the defender
Trade-in of $20,000: Select the challenger

9.19

No option of retention of defender D more than one year


2

AWD = -9000(A/P,10%,1) 192,000


= -9000(1.1000) -192,000
= $-201,900
AWC = -320,000(A/P,10%,4) 68,000 + 50,000(A/F,10%,4)
= -320,000(0.31547) 68,000 + 50,000(0.21547)
= $-158,177
Replace the defender now with System C
9.22

-RV(A/P,8%,3) 60,000 + 15,000(A/F,8%,3) = -80,000(A/P,8%,5)


- [40,000 + 2000(A/G,8%,5)] + 20,000(A/F,8%,5)
-RV(0.38803) 60,000 + 15,000(0.30803) = -80,000(0.25046)
- [40,000 + 2000(1.8465)] + 20,000(0.17046)
-0.38803 RV = 4941.05
RV = $12,734

9.25

-RV(A/P,12%,4) [40,000 + 2000(A/G,12%,4)] = -150,000(A/P,12%,10)


- [10,000 + 500(A/G,12%,10)] + 50,000(A/F,12%,10)
-RV(0.32923) [40,000 + 2000(1.3589)] = -150,000(0.17698)
- [10,000 + 500(3.5847)] + 50,000(0.05698)
-0.32923RV = -35,490 + 42,718
RV = $21,954

9.28

Only 2 options; replace defender, buy challenger now or retain defender for 3 years.
AWD = - (40,000 + 70,000)(A/P,15%,3) 85,000 + 30,000(A/F,15%,3)
= -110,000(0.43798) 85,000 + 30,000(0.28798)
= $-124,538
AWC = -220,000(A/P,15%,3) 65,000 + 50,000(A/F,15%,3)
= -220,000(0.43798) 65,000 + 50,000(0.28798)
= $-146,957
Keep the defender

9.31

Answer is (a)
3

9.34

Answer is (b)

9.37
Years for Years for
Defender Challenger
A
0
3
B
1
2
C
2
1
Option A:
AW = $-73,000
Option

AW, cash flows, $ per year


Year 1
Year 2
Year 3
-73,000
-73,000
-73,000
-74,000
-84,000
-84,000
-74,000
-74,000
-84,000

Option B:

AW = -84,000 + 10,000(P/F,15%,1)(A/P,15%,3)
= -84,000 + 10,000(0.8696)(0.43798)
= $-80,192

Option C:

AW = -74,000 - 10,000(A/F,15%,3)
= -74,000 - 10,000(0.28798)
= $-76,880

Select option A: Replace now.


Answer is (a)

Option
AW, $/year
-73,000
-80,192
-76,880

Solution to every third end-of-chapter problem


Basics of Engineering Economy, 2nd edition
Leland Blank and Anthony Tarquin

Chapter 10
Effects of Inflation
10.1

Inflated dollars are converted into constant value dollars by dividing by one plus
the inflation rate per period for however many periods are involved.

10.4

(a) Cost = 17,131(1 + 0.07)3


= $20,986 per year
(b) Cost = (17,131+ 4000)(1 + 0.07)3
= $25,886 per year

10.7

185(1 + f)5 = 225


(1 + f)5 = 1.2162
1 + f = 1.21620.2
f = 3.99% per year

10.10 (a) Salary in 2017 = 61,872(1.02)6


= $69,678
(b) Salary in 2017 = 61,872(1.07)6
= $92,853
10.13 Transportation cost in 5 years = 7.5(F/P,10%,5)
= 7.5(1.6105)
= 12.08 or 12.1
Labor cost in 5 years = 38.5(F/P,4%,5)
= 38.5(1.2167)
= 46.84 or 46.8
2012: Total cost = 38.5 + 19.5 + 12 + 7.5 + 7 + 6 + 5 + 4.5
= 100
% profit = (4.5/100)(100%)
= 4.50%
2017: Total cost = 46.8 + 19.5 + 12 + 12.1 + 7 + 6 + 5 + 4.5
= 112.9
% profit = (4.5/112.9)(100%)
= 3.99%

10.16

if per month = 24/12 = 2%. Use inflated rate equation to solve for real rate i.
0.020 = i + 0.005 + (i)(0.005)
1.005i = 0.015
i = 0.0149
(1.49% per month)

10.19

if = 0.10 + 0.07 + (0.10)(0.07)


= 17.7%
PW = -150,000 60,000(P/A,17.7%,5) + 0.20(150,000)(P/F,17.7%,5)
= -150,000 - 60,000(3.1485) + 30,000(0.44271)
= $-325,630

10.22

if = 0.12 + 0.04 + (0.12)(0.04) = 16.48%


PWA = 2,100,000(P/F,16.48%,2)
= 2,100,000(0.73705)
= $1,547,806
PWB = 2,100,000 - 400,000 = $1,700,000
The offer from vendor A is better.

10.25 Find F, then deflate the amount by dividing by (1 + f)n


F = 5000(F/P,15%,17) + 8000(F/P,15%,14) + 9000(F/P,15%,13)
+ 15,000(F/P,15%,10) + 16,000(F/P,15%,6) + 20,000
= 5000(10.7613) + 8000(7.0757) + 9000(6.1528)
+ 15,000(4.0456) + 16,000(2.3131) + 20,000
= $283,481
Purchasing power = 283,481/(1.03)17 = $171,511
10.28 (a) F = 10,000(F/P,10%,5)
= 10,000(1.6105)
= $16,105
(b) Purchasing Power = 16,105/(1 + 0.05)5 = $12,619
(c)

if = i + 0.05 + (i)(0.05)
0.10 = i + 0.05 + (i)(0.05)
1.05i = 0.05
i = 0.0476
(4.76%)
or
i = 0.10 0.05/(1 + 0.05) = 0.0476
2

(4.76%)

10.31 Buying power = 1,800,000/(1 + 0.038)20 = $853,740


10.34 Cost in terms of constant-value dollars at real i = 5%
F = 40,000(F/P,5%,3)
= 40,000(1.1576)
= $46,304
10.37

Buying power is P value with inflation removed


P6000 = 6000/(1 + 0.04)2 = $5547
P9000 = 9000/(1 + 0.04)3 = $8001
P5000 = 5000/(1 + 0.04)6 = $3952

10.40

if = 0.15 + 0.05 + (0.15)(0.05)


= 20.75%
AWX = -65,000(A/P,20.75%,5) 40,000
= -65,000(0.33991) 40,000
= $-62,094
AWY = -90,000(A/P,20.75%,5) 34,000 + 10,000(A/F,20.75%,5)
= -90,000(0.33991) 34,000 + 10,000(0.13241)
= $-63,268
Select process X

10.43

if = 0.10 + 0.04 + (0.10)(0.04) = 14.4% per year


AW = -40,000(A/P,14.4%,3) - 24,000 + 6000(A/F,14.4%,3)
= -40,000(0.43363) - 24,000 + 6000(0.28963)
= $-39,607 per year

10.46 Answer is (a)


10.49 Answer is (c)
10.52 Answer is (c)
10.55 if per month = 0.01 + 0.01 + (0.01)(0.01)
= 2.01%
Nominal per year = 2.01(12) = 24.12%
Answer is (c)
3

Solution to every third end-of-chapter problem


Basics of Engineering Economy, 2nd edition
Leland Blank and Anthony Tarquin

Chapter 11
Estimating Costs
11.1

Elements of first cost include equipment cost, delivery charges, installation cost,
insurance coverage, and training of personnel for equipment use.

11.4

Cost = 98.23(50,000) = $4,911,500

11.7

Cost = 1,350,000(1.70/0.93) = $2,467,742

11.10 Cost = 185,000(1634.9/1375.4) = $219,904


11.13 From Table, index value in 2003 = 6695; index value in 2012 = 9290
Ct = 30,000,000(9290/6695)
= $41,628,000
11.16

Cost2011 = Cost2002(1490.2/1104.2)
328 = Cost2002(1.3496)
Cost2002 = $243

11.19

31.39 = Rate1913(16,520.53/100)
Rate1913 = $0.19 per hour

11.22 C2 = 9500(450/250)0.32
= 9500(1.207)
= $11,466
11.25 If C2 = 1.04 C1 and Q2/Q1 = 0.50, the cost-capacity relation is
1.04C1 = C1 (Q2/Q1)x
1.04 = (0.50)x
log 1.04 = x log 0.50
x = -0.0566
11.28 Cost = 3750(2)0.89(1520.6/1104.2) = $9570
11.31

1,320,000 = h(225,000)
h = 5.87

11.34 T12 = T1Ns


s = log 0.85/log 2
= -0.234
T12 = (56)12-0.234
= 31.3 months
11.37 Total direct labor hours = 3000 + 9000 + 5000
= 17,000 hours
Indirect cost/hour = 34,000/17,000
= $2.00
Allocation to Dept A = 3000(2.00)
= $6000
Allocation to Dept B = 9000(2.00)
= $18,000
Allocation to Dept C = 5000(2.00)
= $10,000
11.40 (a) Hand solution: Make alternative indirect cost computation
Rate Usage
Annual cost
(1)
(2)
(3) = (1)(2)
M
$2.40 450,000 $1.08 million
P
$0.50 850,000
425,000
Q
$25
4800
120,000
$/year
$1,625,000
Dept

Determine AW for make and buy alternatives.


AWmake = -3,000,000(A/P,12%,6) + 500,000(A/F,12%,6)
1,500,000 1,625,000
= -3,000,000(0.24323) + 500,000(0.12323) 3,125,000
= $-3,793,075
AWbuy = -4,500,000 200,000(A/G,12%,6)
= -4,500,000 200,000(2.1720)
= $-4,934,400
Select make alternative.

(b) Spreadsheet solution: Select to make inhouse.

11.43 Compare last years allocation based on flight traffic with this years based on
baggage traffic. Significant change took place.
Last year;
This year;
flight basis baggage basis
DFW $330,000
$343,620
YYZ
187,500
218,371
MEX 150,000
105,363
11.46 The answer is (c)
11.49 C2 = 22,000(300/200)0.64 = $28,518
Answer is (b)
11.52 CT = 2.45(390,000) = $955,500
Answer is (a)
11.55

3,000,000 = 550,000(100,000/6000)x
5.4545 = (16.67)x
log 5.4545 = xlog16.67
x = 0.60
Answer is (d)

11.58 Allocation = (900 + 1300)(2000) = $4.4 million


Percent allocated = 4.4/8.0 million = 55%
Answer is (c)

Percent
change
+ 4.1%
+16.5
29.7

Solution to every third end-of-chapter problem


Basics of Engineering Economy, 2nd edition
Leland Blank and Anthony Tarquin

Chapter 12
Depreciation Methods
12.1

Depreciation is a tax-allowed deduction in the equation Taxes = (income


expenses depreciation)(tax rate)

12.4

Productive life Time the asset is actually expected to provide useful service.
Tax recovery period Time allowed by tax laws to depreciate the assets value to
salvage (or zero).
Book recovery period Time used on company accounting books for depreciation
to salvage (or zero)

12.7

Tax depreciation: Dt = Rate(BVt-1)


Book depreciation: Dt = Rate(40,000)
Tax Depreciation
Year, t
Dt
BVt
0
40,000
1
16,000 24,000
2
9,600 14,400
3
5,760
8,640
4
3,456
5,184
Spreadsheet solution with graphs follows.

Book Depreciation
Dt
BVt
40,000
10,000 30,000
10,000 20,000
10,000 10,000
10,000
0

12.10

(a) Depreciation charge is determined from the change in book value


D = 296,000 - 224,000 = 72,000/year
Since the asset has a 5-year life 2 more years of depreciation will reduce
BV to salvage value.
S = 224,000 2(72,000) = $80,000
(b)

(B 80,000)/5 = 72,000
B = $440,000

12.13 Spreadsheet solution uses SL depreciation.


Dt = (350,000 50,000)/5 = $60,000 per year

12.16 Substitute 0.25B for BV. Depreciation rate is d = 2/5 = 0.4


0.25B = B(1 0.4)t
log 0.25 = t(log 0.6)
t = 2.71years
Salvage value will be reached by end of 3rd year
12.19 (a)

SL: BV10 = $10,000 by definition


DDB: Use Equation [12.9] to determine if the implied S < $10,000
with d = 2/7 = 0.2857
BV10 = BV7 = B(1-d)7 = 100,000(0.7143)7
= $9486
Both salvages are less than the market value of $12,500
2

(b)

SL: D10 = (100,000 - 12,500)/10 = $8750


DDB: D10 = 0, since n = 7 years
A spreadsheet solution for both parts follows.

12.22 BV3 = 400,000 - 400,000(0.20 + 0.32 + 0.192)


= $115,200
12.25 SL:

D4 = [80,000 - 0.25(80,000)]/5
= $12,000 per year
BV4 = 80,000 4(12,000)
= $32,000

MACRS: BV4 = 80,000 80,000(0.20 + 0.32 + 0.192 + 0.1152)


= $13,824
Difference = 32,000 13,824 = $18,176
MACRS has a lower BV after 4 years
12.28 MACRS:

D1 = 0.01391(3,400,000) = $47,294
D2 to D10 = 0.02564(3,400,000) = $87,176

Total depreciation for 10 years = 47,294 + 9(87,176) = $831,878


BV10 = 3,400,000 831,878 = $2,568,122
Anticipated selling price is 1.5(BV10) = $3,852,183
Fairfield hopes to sell it for $452,183, or 13.3%, more than they paid for it.

12.31 d = 1/5 = 0.20, For year 2, by DB method


DB : D2 = 0.20(100,000)(1 0.20)1 = $16,000
For SL method, need BV1 from DB method with d = 0.20
BV1 = 100,000(1 0.2)1 = $80,000
New SL depreciation
D2 = (80,000 10,000)/4 = $17,500
SL has larger depreciation by 17,500 16,000 = $1500; switch to SL is advisable
12.34 Verify the following MACRS rates using the modified DDB-to-SL switching.
t
dt

1
0.20

2
0.32

3
0.192

4
0.1152

5
0.1152

6__
0.0576

The DDB rate to start is d = 2/5 = 0.40.


d1:

DDB: d1 = d = 0.20

d2:

Accumulated depreciation = Accumulated D = 0.20


DDB: By Equation [12.14]
d2 = 0.4(1 0.2) = 0.32

(selected)

SL: Modify Equation [12.15] to have denominator (n-t+1.5)


d2 = 0.8/(5-2+1.5) = 0.8/4.5 = 0.178
d3:

Accumulated D = 0.2 + 0.32 = 0.52


DDB: d3 = 0.4(1 0.52) = 0.192
SL:

d4:

(selected)

d3 = 0.48/(5-3+1.5) = 0.137

Accumulated D = 0.2 + 0.32 + 0.192 = 0.712


DDB: d4 = 0.4(1 0.712) = 0.1152
SL:

d4 = 0.288/2.5 = 0.1152

Switch to SL occurs in year 4.


4

(select either)

d5:

Use the SL rate for n = 5


Accumulated D = 0.2 + 0.32 + 0.192+ 0.1152 = 0.8272
SL:

d6:

d5 = 0.1728/(5-5+1.5) = 0.1728/1.5 = 0.1152

d6 is the remainder or 1/2 the d5 rate. Accumulated D = 0.9424


d6 = d5 /2 = 0.0576
or
d6 = 1 0.9424 = 0.0576

12.37

Income = 6000(6) + 7000(9) = $99,000


Percentage depletion charge = 99,000(0.05) = $4950

12.40 (a) Determine the cost depletion factor in $/1000 tons and multiply by yearly
tonnage.
pt = 2,900,000/100 = $29,000 per 1000 tons
Annual cost depletion = volume $29,000
Volume, Cost depletion,
Year 1000 tons
$ per year
1
10
290,000
2
12
348,000
3
15
435,000
4
15
435,000
5
18
522,000
Total
70
$2,030,000
(b) Cost depletion is limited by the total first cost, which is $2.9 million. Since
only $2.03M has been depleted, all 5 years depletion is acceptable.
12.43 Depreciation is same for all years in straight line method.
D3 = [100,000 0.20(100,000)]/5 = $16,000
Answer is (a)
12.46

d = 2/5 = 0.4
BV2 = 30,000(1 0.4)2 = $10,800
Answer is (b)

12.49 Salvage value is always $0 for the MACRS method.


Answer is (a)

12.52 Percentage:

Cost:

GI = 65,000(40) = $2.6 million


Depletion charge = 0.05(2.6 million)
= $130,000
pt = $1.28 per ton
Depletion charge = (65,000)(1.28)
= $83,200

Answer is (b)

Solution to every third end-of-chapter problem


Basics of Engineering Economy, 2nd edition
Leland Blank and Anthony Tarquin

Chapter 13
After-Tax Economic Analysis
13.1

(a) From Table 13.1, marginal tax rate = 39%


(b) Taxes = 0.15(50,000) + 0.25(75,000 50,000) + 0.34(100,000 - 75,000) +
0.39(250,000 100,000)
= $80,750
(c) Average tax rate = (80,750/250,000)(100%) = 32.3%

13.4

TI = 450,000 230,000 48,000 = $172,000


Approximate taxes = 172,000(0.38) = $65,360

13.7

(a) TI = 320,000 149,000 95,000 = $76,000


(b) Use Table 13-1
Taxes = 13,750 + 0.34(76,000-75,000) = $14,090
(c)

Te = 10.5 + (1 0.105)(18.5) = 27.06%


Tax estimate = 76,000(0.2706) = $20,566
Percent of GI = 20,566/320,000 100% = 6.43%

13.10 Before-tax ROR: 0 = -500,000 + 230,000(P/A,i*,5) + 100,000(P/F,i*,5)


i* = 38.48% (spreadsheet)
After-tax ROR = 38.48(1 0.35) = 25.01%
13.13 Missing values are shown in bold red in the table
CFBT2 = 950 150 = $800
D2 = 0.4445(1900) = $845
D4 = 0.0741(1900) = $141
TI3 = 600 200 281 = $119
Taxes2 = -45(0.35) = $-16
CFAT3 = 400 42 = $358
1

Year GI
E
P and S CFBT
0
$-1900 $-1900
1
$800 $-100
0
700
2
950 -150
0
800
3
600 -200
0
400
4
300 -250
700
750

TI

$633 $ 67
845 -45
281 119
141 -91

Taxes
$23
-16
42
-32

CFAT
$-1900
677
816
358
782

13.16 GI can be determined from the data provided.


CFBT = CFAT + taxes
GI E = CFAT + (GI E D)(Te)
Solve for GI to obtain a general relation for each year t.
GIt = [CFAT + Et(1- Te) DtTe]/ (1- Te)
where

CFAT = $2.5 million


Te = 8% + (1-0.08)(20%) = 26.4%
1- Te = 73.6%
(0.736)

(0.264)

Year 1: GI1 = [2.5 million + 650,000(0.736) 650,000(0.264)]/0.736


= $3,813,587
Year 2: GI2 = [2.5 million + 900,000(0.736) 900,000(0.264)]/0.736
= $3,973,913
Year 3: GI3 = [2.5 million + 1,150,000(0.736) 1,150,000(0.264)]/0.736
= $4,134,239
13.19 Total depreciation was 20% + 32% + 19.2% + 11.52% = 82.72%
BV4 = 80,000 80,000(0.8272)
= $13,824 < selling price of $15,000. Depreciation recapture is present
DR = 15,000 13,824
= $1176

13.22 Calculate taxes = GI E (TI + DR)(Te) for each year and add. System 2 has
lower total taxes by $26,600. Spreadsheet solution follows.

13.25 (a) MACRS depreciation


Year
0
1
2
3
4
5
6
7
8
Total

P and
CFBT, $
-200,000
75,000
75,000
75,000
75,000
75,000
75,000
75,000
75,000

Rate

D, $

TI, $

Taxes, $

0.2000
0.3200
0.1920
0.1152
0.1152
0.0576
0
0

40,000
64,000
38,400
23,040
23,040
11,520
0
0

35,000
11,000
36,600
51,960
51,960
63,480
75,000
75,000

13,300
4,180
13,908
19,745
19,745
24,122
28,500
28,500
$152,000

PWtax = 13,300(P/F,8%,1) + 4180(P/F,8%,2) + + 28,500(P/F,8%,8)


= $102,119
(spreadsheet)
Straight line depreciation D = 200,000/8 = $25,000 per year
Taxes = (75,000 25,000)(0.38) = $19,000 per year
Total taxes = 8(19,000) = $152,000
PWtax = 19,000(P/A,8%,8) = 19,000(5.7466)
= $109,185
3

MACRS is preferable with the lower PWtax value.


(b) Total taxes are $152,000 for both methods.
13.28 (a) Defender: Capital loss = BV - Sales price = [300,000 2(60,000)] - 100,000
= $-80,000
Taxes = -80,000(0.35) = $-28,000
This is a tax savings for the challenger in year 0.
CFATC in year 0 = $-420,000 + 28,000 = $-392,000
CFATD in year 0 = $-100,000
(b) Defender:

TI = -120,000 60,000 = $-180,000


Taxes = 180,000(0.35) = $-63,000
CFAT = -120,000 - (-63,000) = $-57,000

Challenger: TI = -30,000 -140,000 = $-170,000


Taxes = -170,000(0.35) = $-59,500
CFAT = -30,000 - (-59,500) = $29,500
(c) AWD = -100,000(A/P,15%,3) 57,000
= -100,000(0.43798) 57,000
= $-100,798
AWC = -392,000(A/P,15%,3) + 29,500
= -392,000(0.43798) + 29,500
= $-142,188
Keep the defender
13.31 The debt portion of $15 million represents 40% of the total.
Total amount of financing = 15,000,000/0.40 = $37,500,000
13.34 Companys equity = 30(0.35) = $10.5 million
Return on Equity = (4/10.5)(100%) = 38.1%

13.37 (a) WACCA = 0.5(9%) + 0.5(6%) = 7.5%


WACCB = 0.2(9%) + 0.8(8%) = 8.2%
Plan A has a lower WACC
(b) Let x = cost of debt capital
WACCA = 8.2% = 0.5(9%) + 0.5x
x = 7.4%
WACCB = 8.2% = 0.2(9%) + 0.8x
x = 8.0%
Plan A cost of debt goes up from 6% to 7.4%; Plan B maintains the same cost.
13.40 (a) Determine the after-tax cost of debt capital and WACC.
After-tax cost of debt capital = 10(1-0.36) = 6.4%
After-tax WACC = 0.65(14.5%) + 0.35(6.4%) = 11.665%
Interest charged to revenue for the project:
14 million 0.11665 = $1,633,100
(b) After-tax WACC = 0.25(14.5%) + 0.75(6.4%)
= 8.425%
Interest charged to revenue for the project:
14 million 0.08425 = $1,179,500
As more and more of the capital is borrowed, the company risks
higher loan rates and owns less and less of itself. Debt capital (loans) will
get more expensive and harder to acquire.

13.43

Find BVt -1 = BV1; solve for NOPAT; find TI from NOPAT; solve for E from TI.
BV1 = 550,000 550,000(0.3333) = $366,685
EVA2 = 28,000 = NOPAT (0.14)(366,685)
NOPAT = $79,336
NOPAT = 79,336 = TI(1 0.35)
TI = $122,055

D2 = 550,000(0.4445)
= $244,475
122,055 = GI - E - D
= 700,000 - E - 244,475
E = $333,470
13.46 Te = 0.08 + (1 0.08)(0.34) = 39.3%
Answer is (b)
13.49

Answer is (d)

13.52

CFAT = GI E TI(Te)
26,000 = 30,000 TI(0.40)
TI = (30,000 26,000)/0.40 = $10,000
Taxes = TI(Te) = 10,000(0.40) = $4000
TI = GI - E D
10,000 = 30,000 D
D = $20,000
Answer is (d)

13.55 BV5 = 100,000(0.0576) = $5760


DR = 22,000 5760 = $16,240
Tax on DR = 16,240(0.30) = $4872
Cash flow = 22,000 4872
= $17,128
Answer is (b)

Solution to every third end-of-chapter problem


Basics of Engineering Economy, 2nd edition
Leland Blank and Anthony Tarquin

Chapter 14
Alternative Evaluation Considering
Multiple Attributes and Risk
14.1

Wi = 1/6 = 0.1667

14.4

(a) Score by attribute with 10 for most important


Logic: #1: 0.90(#5); W1 = 0.90(10) = 9
#2: 0.10(#5); W2 = 0.10(10) = 1
#3: 0.3(#5); W3 = 0.30(10) = 3
#4: 2(#3); W4 = 2(3) = 6
#5: highest score: W5 = 10
#6: 0.8(#4); W6 = 0.80(6) = 4.8
Attribute
1
2
3
4
5
6

Importance
9.0
1.0
3.0
6.0
10.0
4.8
33.8

(b) Normalized weights, Wi = Score/33.8


Attribute
1
2
3
4
5
6
14.7

Wi______
9/33.8 = 0.27
1/33.8 = 0.03
3/33.8 = 0.09
6/33.8 = 0.18
10/33.8 = 0.30
4.8/33.8 = 0.14

Estimates are for the future in engineering economic evaluations. Decision making under
risk is, therefore, always present in any conclusion.

14.10 Determine the probability values for N.


N
P(N)

0
0.12

1
0.56

2
0.26

3
0.032

4_
0 .022

5_
0.006

(a) P(N=0 or 1) = P(N=0) + P(N=1) = 0.12 + 0.56 = 0.68


Percentage is 68% of the households
(b) P(N = 1or 2) =0.56 + 0.26
= 0.82

(82%)

(c) P(N > 3) = P(N=4) + P(N5) = 0.022 + 0.006


= 0.028
(2.8%)
14.13 (a) Use Equation [14.5] to find E(X).
Cell,
Xi, $
600
800
1000
1200
1400
1600
1800
2000

P(Xi)
0.06
0.10
0.09
0.15
0.28
0.15
0.07
0.10
1.00

Xi P(Xi), $
36
80
90
180
392
240
126
200
1344

Sample expected value: E(X) = $1344


(b) Construct a graph with bars at $600, $800, ..., $2000 each at the height of the P(Xi)
value. Show the E(X) = $1344 value on the graph.
14.16 As an example, a sample of 100 values generated the following results:
(a) = AVERAGE(A1:A100) resulted in 49.2532; very close to 50
(b) = STDEV(A1:A100) resulted in 28.08; very close to 28.87
14.19 A simulation similar to Example 14.6 is performed. MARR varies from 7% to 10% with
a 0.25 probability each. Use a lookup table for MARR (columns E and F) coupled with
the RAND() function. For CFAT, use RANDBETWEEN(4000,7000). PW values
(column M) for the simulation shown are positive 23 of the 30 trials.
Conclusion: The project appears economically viable under both certainty and risk.

14.22 Wenv = 50/(100 + 75 + 50) = 50/225 = 0.22


Answer is (c)
14.25 Answer is (c)
14.28 Reading #5 is missing
X = 79 = (81 + 74 + 83 + 66 + x5)/5
395 = 304 + x5
x5 = 91
2 values (66 and 91) are outside the limits
Answer is (b)

Das könnte Ihnen auch gefallen